The Student Room Group

The Proof is Trivial!

Scroll to see replies

Reply 2120
Original post by joostan
OK, I won't. :wink:
Is that problem 161? Looks like a mess, but I might take a punt, right now I'm having a go at some of the others that you just posted :smile:

It's actually really neat and, dare I say, 'straightforward' provided that you have the right tools. The hardest part is having the determination to persist (because everything seems to feel like a dead end in this problem - even if it's along the right lines!)
To demonstrate that these are the only solutions with the given restrictions..

Nice solution bro, but why not simply use an irrationality argument? The question is far more straightforward that its 'BMO2'-status suggests!
Original post by FireGarden
1=limx(1+1x)x=e1^{\infty} = \displaystyle\lim_{x\to\infty} \left(1+\dfrac{1}{x} \right)^{x} = e

It just feels that, representing 11^{\infty} in that way assumes it's indeterminate in the first place as you are adding in the "1+1x1+\frac{1}{x}" which approaches 1 at a different rate to the constant function "1" itself, hence creating a circular argument.
Original post by Smaug123
Here's my "progress post", updated as I solve the question :smile:

k=1 case: I used Mathematica to get a couple of hundred iterations, I just couldn't intuit the limit… then it was easy enough to show by induction that bnan2n>2b_n \equiv \dfrac{a_n^2}{n} > 2 always, after getting a recurrence relation for bnb_n. Also shown previously that bnb_n is strictly decreasing. Now I just need to show that 2 is indeed the limit.

I wouldn't get too caught up in pedantry! My solution came out of what I initially considered to be a conjecture until I rigorously formalized my proof. :tongue:
Original post by MW24595
Solution 307

Solution 310

\displaystyle \frac{1}{2} \displaystyle \sum_{m=1}^{\infty} \sum_{n=1}^{\infty} \left( \frac{m^2 n(3^n m)+ n^2 m (3^m n)}{3^{m+n} (n 3^m + m3^n)} \right)

Awesome solutions bro, I did the same in both cases. Also, this line in a typo^ :tongue:
Original post by Jkn
It's actually really neat and, dare I say, 'straightforward' provided that you have the right tools. The hardest part is having the determination to persist (because everything seems to feel like a dead end in this problem - even if it's along the right lines!)

Nice solution bro, but why not simply use an irrationality argument? The question is far more straightforward that its 'BMO2'-status suggests!

OK, I'll give it a go tomorrow.
I figured irrationality looked like a faff of writing words, to be honest the quadratic looks messy but is actually quite simple :tongue:
I've spent a while on Problem 305.

Spoiler

Reply 2122
Original post by joostan
OK, I'll give it a go tomorrow.
I figured irrationality looked like a faff of writing words, to be honest the quadratic looks messy but is actually quite simple :tongue:
I've spent a while on Problem 305.

Spoiler


Spoiler

Reply 2123
Original post by Jkn

Problem 308
*

Find all pairs of integers (x,y) such that 1+x2y=x2+2xy+2x+y\displaystyle 1+x^2y=x^2+2xy+2x+y

Problem 314*

Find all positive integers n such that n+2012n+2012 divides n2+2012n^2+2012 and n+2013n+2013 divides n2+2013n^2+2013.



Solution 308

The first thing one should note, is that, considering the equation  mod x \ mod \ x , we have, y1 mod xy10 mod x y \equiv 1 \ mod \ x \Rightarrow y-1 \equiv 0 \ mod \ x. This is, ofcourse, considering that x0 x \not= 0 .

If it is, then simple substitution gives us, y= 1. So, (0,1) (0,1) is one solution. Let us now assume that x is non-zero.

Now,

1+x2y=x2+2xy+2x+y(x21)(y1)=2x(y+1)(x1)(x+1)(y1)=2x(y+1) 1+x^2 y = x^2+ 2 xy+ 2x + y \Rightarrow (x^2-1)(y-1) = 2x (y+1) \Rightarrow (x-1)(x+1)(y-1)= 2x (y+1) .

As, xy1 x | y-1 , let y1=kx y-1 = kx for some, kZ k \in \mathbb{Z} .

kx(x1)(x+1)=2x(kx+2)k(x1)(x+1)=2(kx+2)[br][br]kx22kxk4=0[br][br]x=2±4k2+4k(k+4)2k=1±1+k+4k[br][br]kk+4k4k=±1,±2,±,4[br] \Rightarrow kx(x-1)(x+1) = 2x (kx + 2) \Rightarrow k(x-1)(x+1) = 2(kx+2)[br][br]\Rightarrow kx^2 - 2kx - k - 4 = 0[br][br]\Rightarrow x=\displaystyle \frac{2 \pm \sqrt{4k^2 + 4k(k+4)}}{2k} = 1 \pm \sqrt{1+ \frac{k+4}{k}}[br][br]\Rightarrow k| k+4 \Rightarrow k|4 \Rightarrow k = \pm 1, \pm 2, \pm, 4[br]

Substituting these into the expression for x x, we see that only k=4,2,2 k = -4, 2, 2 yield integer values for xx .

So, we have possible pairs, (k,x)=(4,0),(4,2),(2,1),(2,1),(2,3) (k,x) = (-4, 0), (-4, 2), (-2, 1), (2, -1), (2, 3) .

Computing y y from these, we have, (x,y)=(0,1),(2,7),(1,1),(3,7) (x, y)= (0,1), (2, -7), (-1, 1), (3, 7) , and, these are the only possible pairs of integers satisfying the equation above.

Solution 314

I'm posting this, because I think mine's one way of looking at it, but I'm concealing it because it's a nice one that I hope others solve as well.

Spoiler

.
(edited 10 years ago)
Reply 2124
Original post by Jkn

Problem 305*

Prove that the sequence defined by y0=1y_0=1, yn+1=12(3yn+5yn24)\displaystyle y_{n+1}=\frac{1}{2} \left(3y_n + \sqrt{5y_n^2-4} \right) for n0n \ge 0 consists only of integers.

[As a bonus, can you find any 'similar' sequences for which this is true?]



I'll wait for others to post a solution to this one first, but I just wanted to point out something.

Did anybody else notice that yn=F2n+1 y_n = F_{2n+1} , where Fn F_n denotes the nth Fibonacci number? :tongue:
So, this sequence, essentially generates every other Fibonacci number.

Remember that as n,Fn+11+52Fn n \to \infty, \displaystyle F_{n+1} \cong \frac{1+ \sqrt {5}}{2} F_n .

Now, futher note that, as, n,yn+13+52yn n \to \infty, \displaystyle y_{n+1} \cong \frac{3+ \sqrt{5}}{2} y_n .

And finally, notice that, 3+52=(1+52)2 \displaystyle \frac{3+ \sqrt{5}}{2} = \left(\frac{1+ \sqrt {5}}{2} \right)^2 . :colone:

Original post by Jkn
Awesome solutions bro, I did the same in both cases. Also, this line in a typo^ :tongue:


Cheers, man. :tongue: These were a good bunch, eh?
(edited 10 years ago)
Original post by Jkn

Problem 312*

Evaluate 4421+42441+44481+484161+\displaystyle \frac{4}{4^2-1}+\frac{4^2}{4^4-1}+\frac{4^4}{4^8-1}+\frac{4^8}{4^{16}-1}+\cdots



An ugly solution, but a solution nonetheless.

Let Sn(x)=xx21+x2x41+x4x81+x8x161+...+x2nx2n+11 \displaystyle S_n(x) = \frac{x}{x^2-1}+\frac{x^2}{x^4-1}+\frac{x^4}{x^8-1}+\frac{x^8}{x^{16}-1} + ... + \frac{ x^{2^n} }{ x^{2^{n+1}} -1}

After adding a couple of terms I guessed that (and proved it by induction)

Sn(x)=x+x2+x3+...+x2n+11x2n+11\displaystyle S_n(x) = \frac{x+x^2+x^3+...+x^{2^{n+1}-1}}{x^{2^{n+1}}-1}

which can be written as Sn(x)=xx11x1x2n+111x2n+1\displaystyle S_n(x) = \frac{x}{x-1} \cdot \frac{\frac{1}{x}-\frac{1}{x^{2^{n+1}}}}{1-\frac{1}{x^{2^{n+1}}}} by summing the GP and tidying up a little.

Taking the limit, we get limnSn(x)=1x1 \displaystyle \lim_{n \rightarrow \infty} S_n(x)=\frac{1}{x-1}

So for this question setting x=4x=4 gives the sum as 13\frac{1}{3}.
(edited 10 years ago)
Reply 2126
Original post by Jkn

Problem 312*

Evaluate 4421+42441+44481+484161+\displaystyle \frac{4}{4^2-1}+\frac{4^2}{4^4-1}+\frac{4^4}{4^8-1}+\frac{4^8}{4^{16}-1}+\cdots


Solution 312


4421+42441+44481+484161+=14(1+142+144+...)+142(1+144+148+...)+144(1+148+1416+...)+... \displaystyle \frac{4}{4^2-1}+ \frac{4^2}{4^4-1}+ \frac{4^4}{4^8-1}+\frac{4^8}{4^{16}-1}+ \cdots= \frac{1}{4} \left(1+ \frac{1}{4^2}+ \frac{1}{4^4}+... \right)+ \frac{1}{4^2} \left(1+ \frac{1}{4^4}+ \frac{1}{4^8}+... \right)+\frac{1}{4^4} \left(1+ \frac{1}{4^8}+ \frac{1}{4^{16}}+... \right)+...
=(14+143+145+...)+(142+146+1410+...)+(144+1412+1420+...)+... \displaystyle = \left(\frac{1}{4}+ \frac{1}{4^3}+ \frac{1}{4^5}+... \right)+ \left(\frac{1}{4^2}+ \frac{1}{4^6}+ \frac{1}{4^{10}}+... \right)+ \left(\frac{1}{4^4}+ \frac{1}{4^{12}}+ \frac{1}{4^{20}}+... \right)+...

Note that all (negative) exponents of 44 in the first bracket are 1mod2\equiv 1 \mod 2 (moreover, they contain all positive integers of this form), the next bracket has terms 2mod4\equiv 2 \mod 4, then the third bracket 4mod8\equiv 4 \mod 8 ...

Lemma: All positive integers are of the form 2n1mod2n\equiv 2^{n-1} \mod 2^{n} for some unique nn, (proof of n's existence and uniqueness taken to be obviously true, can clarify if unclear).

As there must exist exactly a single bracket containing the negative exponents of the form 2n1mod2n\equiv 2^{n-1} \mod 2^{n}, and there exist brackets for each nn, all negative exponents but zero exist in the expansion.

Thus the expansion is equal to
1+14+142+143+...=11141=131+\frac{1}{4}+\frac{1}{4^2}+ \frac{1}{4^3} +...= \frac{1}{1-\frac{1}{4}}-1=\frac{1}{3}
(edited 10 years ago)
Reply 2127
Original post by MW24595
Solution 308

Looks a tad longer than I had expected. Don't think that was quite how I did it, but good none the less! :smile:
Solution 314

I'm posting this, because I think mine's one way of looking at it, but I'm concealing it because it's a nice one that I hope others solve as well.

Again, not how I did it though just as neat. I think there's no surprise you went for divisibility/modular-arithmetic-style solutions whilst I went for good old fashioned algebra in both cases (I should really start trying to produce some solutions more like yours as there may well be cases where your style proves more generalisable). :tongue:
Original post by MW24595

Spoiler



Cheers, man. :tongue: These were a good bunch, eh?

Nice use of spoiler brackets there! :lol: I'm pretty sure that's the key to the problem? There are also several methods that could have been used to spot that. How did you spot it?

Yeah they are! Some I found rather boring (mostly the BMO1/BMO2 ones as they are often rather trivial and rely on the application of a single technique - one that cannot be known without the specialist knowledge - which kind of frustrates me that I never had the chance to do one of these competitions since I have learnt the techniques :lol:) The one above is fun though, as are all the non-BMO questions (in my opinion)!

Oh and, btw, I eagerly await your solution to the an+1=an+1ank\displaystyle a_{n+1}=a_{n}+\frac{1}{\sqrt[k]{a_{n}}} question... :colone:
Original post by bogstandardname
An ugly solution, but a solution nonetheless.

Original post by henpen
Solution 312

Nice stuff guys, it's great to see a lot of alternative solutions! :biggrin:

For me, as a lover of direct algebraic solutions, this method simply jumped out at me.

Solution 312 (3)

Factorising the denominator, using the method of differences and then noting that the nth term tends to zero, we have:

n=0(42n+1)1(42n+1)(42n1)=n=0(142n1142n+11)=14201=13 \displaystyle \begin{aligned} \sum_{n=0}^{\infty} \frac{(4^{2^{n}}+1)-1}{(4^{2^{n}}+1)(4^{2^{n}}-1)} = \sum_{n=0}^{\infty} \left( \frac{1}{4^{2^{n}}-1} - \frac{1}{4^{2^{n+1}}-1} \right) = \frac{1}{4^{2^{0}}-1}=\frac{1}{3} \ \square \end{aligned}
Original post by james22
Problem 304*

Does there exist an integer, k, such that using only at most k coin flips, a number in the set {1,2,3} can be randomly picked with each being equally likely?


Original post by Jkn
Solution 304

There exists no such integer k....


If the coin is thick enough to flip heads, tails and side with equal probability then clearly k = 1. :tongue: Thanks to some physicists from Harvard, one knows exactly how to construct such a coin.
Original post by ukdragon37
If the coin is thick enough to flip heads, tails and side with equal probability then clearly k = 1. :tongue: Thanks to some physicists from Harvard, one knows exactly how to construct such a coin.


PRSOM - this is an excellent solution!
Original post by MW24595
I'll wait for others to post a solution to this one first, but I just wanted to point out something.

Did anybody else notice that yn=F2n+1 y_n = F_{2n+1} , where Fn F_n denotes the nth Fibonacci number? :tongue:
So, this sequence, essentially generates every other Fibonacci number.

Yep. I did :tongue:


Original post by Jkn

Problem 305*

Prove that the sequence defined by y0=1y_0=1, yn+1=12(3yn+5yn24)\displaystyle y_{n+1}=\frac{1}{2} \left(3y_n + \sqrt{5y_n^2-4} \right) for n0n \ge 0 consists only of integers.

[As a bonus, can you find any 'similar' sequences for which this is true?]


Solution 305:
Lemma: (I hope that's used correctly :tongue:)
Fn2Fn+1Fn1=(1)n1F_n^2-F_{n+1}F_{n-1}=(-1)^{n-1}
Where FnF_n denotes the nthn^{th} Fibonacci number.
This is Cassini's Identity.
Proof:
Basis Case: F12F2F0=1=(1)0F_1^2-F_2F_0=1=(-1)^0
Now assume that the identity holds for:
n=k kNn=k \ k \in \mathbb{N} So that Fk2Fk+1Fk1=(1)k1F_k^2-F_{k+1}F_{k-1}=(-1)^{k-1}
Now consider;
Fk+12FkFk+2=Fk+12Fk(Fk+1+Fk)=Fk+12FkFk+1Fk2[br]Fk+12FkFk+2=Fk+1(Fk+Fk1)FkFk+1Fk2=Fk+1Fk1Fk2F_{k+1}^2-F_kF_{k+2}=F_{k+1}^2-F_k(F_{k+1}+F_{k})=F_{k+1}^2-F_kF_{k+1}-F_k^2[br]\Rightarrow F_{k+1}^2-F_kF_{k+2}=F_{k+1}(F_k+F_{k-1})-F_kF_{k+1}-F_k^2=F_{k+1}F_{k-1}-F_k^2
Recall that: Fk2Fk+1Fk1=(1)k1F_k^2-F_{k+1}F_{k-1}=(-1)^{k-1}
Fk+12FkFk+2=(1)(1)k1=(1)k\Rightarrow F_{k+1}^2-F_kF_{k+2}=(-1)(-1)^{k-1}=(-1)^k
This completes the induction.

Now to the problem:
yn+1=12(3yn+5yn24)y_{n+1}=\dfrac{1}{2} \left(3y_n + \sqrt{5y_n^2-4} \right)
y0=1(=F1)[br]y1=2 (=F3)[br]y2=5 (=F5)[br]y3=13 (=F7)y_0=1 (=F_1)[br]y_1=2 \ (=F_3)[br]y_2=5 \ (=F_5)[br]y_3=13 \ (=F_7)
This is looking a lot like alternate Fibonacci numbers.
So the proposition is that: yn=F2n+1y_n=F_{2n+1}
We have the Basis Case(s) for n={0,1,2,3}n=\{0,1,2,3\}
So assume that this is true for n=k, kNn=k, \ k \in \mathbb{N}
So that: yk=F2k+1y_k=F_{2k+1}
Now:
yk+1=12(3F2k+1+5F2k+124)y_{k+1}=\dfrac{1}{2} \left(3F_{2k+1} + \sqrt{5F_{2k+1}^2-4} \right)
Consider:
5F2k+124\sqrt{5F_{2k+1}^2-4}
Observing that by the lemma:
4=(4)(1)2k=4(F2k+12F2kF2k+2)[br]5F2k+124=5F2k+124(F2k+12F2kF2k+2)[br]5F2k+124=F2k+12+4F2k(F2k+1+F2k)[br]5F2k+124=F2k+12+4F2kF2k+1+4F2k2[br]5F2k+124=(F2k+1+2F2k)2=(F2k+1+2F2k)-4=(-4)(-1)^{2k}=-4(F_{2k+1}^2-F_{2k}F_{2k+2})[br]\Rightarrow \sqrt{5F_{2k+1}^2-4}=\sqrt{5F_{2k+1}^2-4(F_{2k+1}^2-F_{2k}F_{2k+2})}[br]\Rightarrow \sqrt{5F_{2k+1}^2-4}=\sqrt{F_{2k+1}^2+4F_{2k}(F_{2k+1}+F_{2k})}[br]\Rightarrow \sqrt{5F_{2k+1}^2-4}=\sqrt{F_{2k+1}^2+4F_{2k}F_{2k+1}+4F_{2k}^2}[br]\Rightarrow \sqrt{5F_{2k+1}^2-4}=\sqrt{(F_{2k+1}+2F_{2k})^2}=(F_{2k+1}+2F_{2k})
Now substituting this in to the recurrence relation yields:
yk+1=12(3F2k+1+F2k+1+2F2k)=2F2k+1+F2ky_{k+1}=\dfrac{1}{2} \left(3F_{2k+1}+F_{2k+1}+2F_{2k} \right)=2F_{2k+1}+F_{2k}
yk+1=(F2k+1+F2k)+F2k+1=F2k+2+F2k+1[br]yk+1=F2k+3\Rightarrow y_{k+1}=(F_{2k+1}+F_{2k})+F_{2k+1}=F_{2k+2}+F_{2k+1}[br]\therefore y_{k+1}=F_{2k+3}
This implies that all yn=F2n+1 nZ+y_n=F_{2n+1} \ \forall n \in \mathbb{Z^+}
Since every term in the Fibonacci sequence is an integer (since an integer added to another integer is itself an integer) it follows that every term in the sequence:
yn+1=12(3yn+5yn24)y_{n+1}=\dfrac{1}{2} \left(3y_n + \sqrt{5y_n^2-4} \right)
Is also an integer.

Bonus:

Spoiler

(edited 10 years ago)
Original post by ukdragon37
If the coin is thick enough to flip heads, tails and side with equal probability then clearly k = 1. :tongue: Thanks to some physicists from Harvard, one knows exactly how to construct such a coin.


Nice, but by coin I really meant random binary generator.
Reply 2132
Problem 316*

Prove that
j<k,j,kN1j2k2=π4120\displaystyle \sum_{j<k, j,k \in \mathbb{N}}\frac{1}{j^2k^2}= \frac{\pi^4}{120}
and find a general form for
x1<x2<...<xn,x1,x2,...,xnN1k=1nxk2\displaystyle \sum_{x_1<x_2<...<x_n, x_1,x_2,...,x_n \in \mathbb{N}}\frac{1}{\prod_{k=1}^nx_k^2}.

Note that I'm taking the set of natural numbers to not include 0.

Spoiler

(edited 10 years ago)
Reply 2133
Original post by henpen
Problem 316*

Prove that
j<k,j,kN1j2k2=π4120\displaystyle \sum_{j<k, j,k \in \mathbb{N}}\frac{1}{j^2k^2}= \frac{\pi^4}{120}
and find a general form for
x1<x2<...<xn,x1,x2,...,xnN1k=1nxk2\displaystyle \sum_{x_1<x_2<...<x_n, x_1,x_2,...,x_n \in \mathbb{N}}\frac{1}{\prod_{k=1}^nx_k^2}


Solution 316

Notice that,

j<k,j,kN1j2k2=i=11i2j=i+11j2 \displaystyle \sum_{j<k, j,k \in \mathbb{N}}\frac{1}{j^2k^2} = \sum_{i=1}^{\infty} \frac{1}{i^2} \sum_{j=i+1}^{\infty} \frac{1}{j^2} .

Now, note that:

sinyy=r=1(1)ryr2r+1! \displaystyle \frac{\sin {\sqrt {y}}}{\sqrt{y}} = \displaystyle \sum_{r=1}^{\infty} (-1)^r \frac{y^r}{2r+1!} .

Consider the Weierstrauss Product of this function, i.e, when a function is expressed as a product of linear factors, each of which has a root equal to the root of this function.

Now, sinyy \displaystyle \frac{\sin {\sqrt {y}}}{\sqrt{y}} has roots at ±π,±2π,±3π... \pm \pi, \pm 2 \pi, \pm 3 \pi ... .

sinyy=(1yπ)(1+yπ)(1y2π)...[br][br]sinyy=i=1(1yi2π2)[br][br] \Rightarrow \displaystyle \frac{\sin {\sqrt {y}}}{\sqrt{y}} = (1- \frac{\sqrt y}{\pi})(1+ \frac{\sqrt y}{\pi})(1- \frac{\sqrt y}{2 \pi})...[br][br]\Rightarrow \displaystyle \frac{\sin {\sqrt {y}}}{\sqrt{y}} = \displaystyle \prod_{i=1}^{\infty} \left(1- \frac{y}{i^2 \pi ^2} \right)[br][br].

And herein lies the crux move. Consider the co-efficient of y2 y^2 in this expansion.

We have:

Co-efficient of y2=1π4((122+132...)+122(132+142...)...)=1π4i=11i2j=i+11j2 y^2 = \displaystyle \frac{1}{\pi^4} \left( \left(\frac{1}{2^2}+ \frac{1}{3^2}... \right)+ \frac{1}{2^2} \left(\frac{1}{3^2}+ \frac{1}{4^2}...\right) ... \right) = \frac{1}{\pi^4} \sum_{i=1}^{\infty} \frac{1}{i^2} \sum_{j=i+1}^{\infty} \frac{1}{j^2} .

And voila, this is (almost) the very expression we had in the first line.

Notice that in the Maclaurin expansion of sinyy \displaystyle \frac{\sin {\sqrt {y}}}{\sqrt{y}} , we had the co-efficient of y2=15! y^2 = \frac{1}{5!} .

Equating the 2, we have:

1π4i=11i2j=i+11j2=15![br][br]i=11i2j=i+11j2=π45![br][br]j<k,j,kN1j2k2=π45! \displaystyle\frac{1}{\pi^4} \sum_{i=1}^{\infty} \frac{1}{i^2} \sum_{j=i+1}^{\infty} \frac{1}{j^2} = \frac{1}{5!}[br][br]\Rightarrow \displaystyle \sum_{i=1}^{\infty} \frac{1}{i^2} \sum_{j=i+1}^{\infty} \frac{1}{j^2} = \frac{\pi^4}{5!}[br][br]\Rightarrow \displaystyle \sum_{j<k, j,k \in \mathbb{N}}\frac{1}{j^2k^2} = \frac{\pi^4}{5!}

A generalization can easily be undertaken considering higher powers of y y in both expressions.

So, we have,

x1<x2<...<xn,x1,x2,...,xnN1k=1nxk2=π2n2n+1! \displaystyle \sum_{x_1<x_2<...<x_n, x_1,x_2,...,x_n \in \mathbb{N}}\frac{1}{\prod_{k=1}^nx_k^2} = \displaystyle \frac{\pi^{2n}}{2n+1!}
Reply 2134
Original post by MW24595
Solution 316
...
And herein lies the crux move. Consider the co-efficient of y2 y^2 in this expansion.

We have:

Co-efficient of y2=1π4((122+132...)+122(132+142...)...)=1π4i=11i2j=i+11j2 y^2 = \displaystyle \frac{1}{\pi^4} \left( \left(\frac{1}{2^2}+ \frac{1}{3^2}... \right)+ \frac{1}{2^2} \left(\frac{1}{3^2}+ \frac{1}{4^2}...\right) ... \right) = \frac{1}{\pi^4} \sum_{i=1}^{\infty} \frac{1}{i^2} \sum_{j=i+1}^{\infty} \frac{1}{j^2} .

And voila, this is (almost) the very expression we had in the first line.

Notice that in the Maclaurin expansion of sinyy \displaystyle \frac{\sin {\sqrt {y}}}{\sqrt{y}} , we had the co-efficient of y2=15! y^2 = \frac{1}{5!} .

Equating the 2, we have:

1π4i=11i2j=i+11j2=15![br][br]i=11i2j=i+11j2=π45![br][br]j<k,j,kN1j2k2=π45! \displaystyle\frac{1}{\pi^4} \sum_{i=1}^{\infty} \frac{1}{i^2} \sum_{j=i+1}^{\infty} \frac{1}{j^2} = \frac{1}{5!}[br][br]\Rightarrow \displaystyle \sum_{i=1}^{\infty} \frac{1}{i^2} \sum_{j=i+1}^{\infty} \frac{1}{j^2} = \frac{\pi^4}{5!}[br][br]\Rightarrow \displaystyle \sum_{j<k, j,k \in \mathbb{N}}\frac{1}{j^2k^2} = \frac{\pi^4}{5!}

...


Nice. Consider this way as an alternative to your expansion:

sin(y)y=k=0akyk\displaystyle \frac{\sin(\sqrt{y})}{\sqrt{y}}=\sum_{k=0}^\infty a_k y^k

Letting rkr_k be the roots of the function, we have
a0=(1)nk=0rk=1\displaystyle a_0=(-1)^n\prod_{k=0}^\infty r_k=1
a1=(1)n1k=11rkk=0rk=13!\displaystyle a_1=(-1)^{n-1}\sum_{k=1}^\infty \frac{1}{r_k}\prod_{k=0}^\infty r_k=\frac{-1}{3!}
a1=(1)n1j,k1rkrjk=0rk=15!\displaystyle a_1=(-1)^{n-1}\sum_{j,k}^\infty \frac{1}{r_kr_j}\prod_{k=0}^ {\infty} r_k=\frac{1}{5!}
, where nn is an arbitrary integer (basically aka_k and akla_{k-l} have opposite parity only if ll is odd).
Then take the quotient ama0\frac{a_m}{a_0} and substitute rkr_k to get the required result.
(edited 10 years ago)
Reply 2135
Problem 317**


Find

k=12k+12π(2k+1)2\displaystyle \sum_{k=-\infty}^{\infty}\frac{1}{2k+1}-\frac{2}{\pi(2k+1)^2}
Reply 2136
Original post by henpen
Problem 317**


Find

k=12k+12π(2k+1)2\displaystyle \sum_{k=-\infty}^{\infty}\frac{1}{2k+1}-\frac{2}{\pi(2k+1)^2}


I'm not quite sure this is right, but...

Solution 317 (I think?)

Spoiler

(edited 10 years ago)
Original post by MW24595
Solution 317 (I think?)

Does this not require absolute convergence? You've split up the summand, which you can't necessarily do without absolute convergence.
Reply 2138
Original post by Smaug123
Does this not require absolute convergence? You've split up the summand, which you can't necessarily do without absolute convergence.


Hence my hesitation. In fact, I don't think it is.(?)
Original post by MW24595
Hence my hesitation. In fact, I don't think it is.(?)

I'm not sure - it feels not-absolutely-convergent, but I don't know. Mathematica can't make heads or tails of the absolute series - refuses to even give me a numerical approximation.

Quick Reply

Latest